0

I've been trying to solve this problem, I realized the following: For example, for $3|6$ we have:

$$F_6=F_5+F_4=(F_4+F_3)+F_4=2F_4 + F_3$$

I realized that doing this, for $m,n$ and $m|n$ we will always get an expression such as:

$$F_n=xF_m+yF_{m+1}$$

The value of $x$ doesn't seems to matter. But if $y=F_m$ which I strongly suspect it is, then we are done because:

$$F_n=F_m(x+F_{m+1})$$

Until now, I've been unable to show that $y=F_m$ or find another way to prove this. I've been stuck for a while, can you help?

Red Banana
  • 23,956
  • 20
  • 91
  • 192
  • 4
    Nice observation! Yes, a result of this form is true and you can prove it by induction. If you know some modular arithmetic you can think about computing the Fibonacci sequence mod a Fibonacci number. – Qiaochu Yuan Oct 06 '20 at 06:37
  • 1
    Read up this post : https://math.stackexchange.com/questions/11477/fibonacci-addition-law-f-nm-f-n-1f-m-f-n-f-m1?rq=1 , and see if the pattern you are referring to is reflected or not. Some of the answers there present generalizations that you might enjoy as well, including a combinatorial proof, and a proof via matrices. – Sarvesh Ravichandran Iyer Oct 06 '20 at 06:48
  • 1
    Also: https://math.stackexchange.com/q/378116/42969. – Martin R Oct 06 '20 at 06:55

1 Answers1

1

Hint: At the bottom of the wikipedia article on the matrix form of the Fibonacci sequence are reported the relations

\begin{align} {F_m}{F_n} + {F_{m-1}}{F_{n-1}} &= F_{m+n-1},\\ F_{m} F_{n+1} + F_{m-1} F_n &= F_{m+n} . \end{align} In particular, with $m=n$, \begin{align} F_{2n-1} &= F_n^2 + F_{n-1}^2\\ F_{2n} &= (F_{n-1}+F_{n+1})F_n\\ &= (2F_{n-1}+F_n)F_n . \end{align} Using these with $m=k\cdot n$ so that $m+n=(k+1)n$ and induction will lead you to the result.


Full proof:

We want to show that if $m\mid n$ then $F_m\mid F_n$.

Now, $m\mid n$ is equivalent to showing that $n=km\ \ k\in\mathbb{N}$. The induction will be over $k$.

Obviously $k=1$ ($m=n$) is true.

Now let's assume the statement is true for $1\leq k\leq K-1$, let's show it is true for $n=mK$ as well. By the second equality, choosing $n=(K-1)m$ we obtain

\begin{align*} F_{n}=F_{Km}=F_{m+(K-1)m}&=F_{m}F_{(K-1)m+1}+F_{m-1}F_{(K-1)m}. \end{align*} But now by induction hypothesis $F_{(K-1)m}$ is divisible by $F_m$ thus $F_{(K-1)m}=F_m\cdot d$ and hence

\begin{align*} F_{n}=F_{m}F_{(K-1)m+1}+F_{m-1}F_m\cdot d=F_m\cdot (F_{(K-1)m+1}+F_{m-1}d). \end{align*} which shows that $F_m\mid F_n$

b00n heT
  • 16,360
  • 1
  • 36
  • 46